LSAT and Law School Admissions Forum

Get expert LSAT preparation and law school admissions advice from PowerScore Test Preparation.

 ay514
  • Posts: 27
  • Joined: Aug 02, 2011
|
#1806
Hello,

I'm doing PrepTest 38 (October 2002) and was wondering if you could provide me with some insight as I'm having a lot of difficulty with this question. The question starts with People who do not believe that others distrust them...I diagrammed all the conditionals and got:

~believe others distrust them --> confident
tend to trust others --> regard difficult task as challenge
confident --> regard difficult tasks as challenge

but from there I'm somewhat stuck...I don't see any way to connect the conditionals except ~believe others distrust them --> confident --> regard difficult tasks as challenge

which I don't think helps me solve the question. Any help would be great! Thank you!
User avatar
 Dave Killoran
PowerScore Staff
  • PowerScore Staff
  • Posts: 5972
  • Joined: Mar 25, 2011
|
#1811
In Justify questions, you have to isolate the conclusion, because that's the statement being produced by the combination of the premises and the correct answer choice. In this case, the conclusion is in the middle of the stimulus:

Premise: ~believe others distrust them --> confident
Premise: confident --> regard difficult tasks as challenge
(these two combine as: ~believe others distrust them --> confident --> regard difficult tasks as challenge )

Conclusion: tend to trust others --> regard difficult task as challenge

So, now connect the "~believe others distrust them" and the "tend to trust others" and you get (C).

In abstract form, this argument appears as:

Premise: B --> C
Premise: C --> D
Conclusion: A --> D

Answer choice (C): A --> B

Does that make sense? Please let me know. Thanks!
 ay514
  • Posts: 27
  • Joined: Aug 02, 2011
|
#1821
Yes, it does, especially the last part with the letters as simple explanations. Thank you so much!! You're always a great help!
 desmail
  • Posts: 50
  • Joined: Jul 05, 2011
|
#3671
I know why C is correct, but why isnt the answer B?

If our premises are: ~beleive others distrust them-->confident abilities-->challenge

and our conclusion is: trust others-->challenge

Why cant we say confident abilities-->trust others--> challenge?

This would still lead us to our trust others-->challenge conclusion, right?
User avatar
 Dave Killoran
PowerScore Staff
  • PowerScore Staff
  • Posts: 5972
  • Joined: Mar 25, 2011
|
#3675
Hey Dana,

No, it doesn't work actually. It look like you are treating the conclusion as a premise that can be added back into the mix.

The goal in this question is to add a statement to the premises only that will result in the conclusion.

Using the A, B, C, D, model above, we have:

Premises: B --> C --> D
Conclusion: A --> D

Looking at (B) in this fashion, you have it diagrammed as C --> A. So, let's look at that in conjunction with just the premises:

B --> C --> A and D

Does that result in A --> D? No, unfortunately, it doesn't, and so (B) is incorrect.

Please let me know if that helps. Thanks!
 desmail
  • Posts: 50
  • Joined: Jul 05, 2011
|
#3678
Hi Dave,

So are you saying:

If we have B-->C-->D

Conc: A-->D

We need a premise that will make A-->D happen. If we have B-->C-->A, this doesn't guarentee that A-->D will happen because only C is linked to D?

Is the gap we are trying to figure out between A ?? D or is the gap between B-->C-->D and the conclusion?

Is it kind of like saying if all C's are D's, and if all C's are A's, it doesnt necessarily mean A's and D's overlap?

Thank you!
User avatar
 Dave Killoran
PowerScore Staff
  • PowerScore Staff
  • Posts: 5972
  • Joined: Mar 25, 2011
|
#3686
Hi Dana,

Yes, you pretty much have it at this point!
desmail wrote:Hi Dave,
If we have B-->C-->D

Conc: A-->D

We need a premise that will make A-->D happen. If we have B-->C-->A, this doesn't guarentee that A-->D will happen because only C is linked to D?
Correct. when you take this approach, C is linked to both A and D, but they are both necessary conditions of C, and aside from that fact, no direct connection exists between them. Analogically speaking, it's like this:

To get into Harvard, you must be both rich and famous.

Does that mean that if you are rich, then you are automatically famous? No, and thus we can't conclude A --> D here.
desmail wrote: Is the gap we are trying to figure out between A ?? D or is the gap between B-->C-->D and the conclusion?

Is it kind of like saying if all C's are D's, and if all C's are A's, it doesnt necessarily mean A's and D's overlap?
Thank you!
Yes, exactly :D

And also, the gap is between the premises and conclusion. In Justify questions you need to add something (the correct answer choice) to the premises that will force the conclusion to be produced. that's why we can take the premises, add answer choice (B) to them, and then see if we get the conclusion. We don't, and that's why (B) is incorrect.

It looks like you have it nailed down now, but let me know if I can clarify anything. Thanks!
 voodoochild
  • Posts: 185
  • Joined: Apr 25, 2012
|
#4802
I have no doubt about the OA = C. But, my question is more about the conditional logic.

Let's first dissect the argument:

Premise - People who don't believe that others distrust them => confident
Premise - People who are confident => regard such as X

Conclusion - People who tend to distrust others regard as X...

The BIG gap here is about "People who don't believe that others distrust them" and "People who tend to distrust others"

My question is that :

the correct answer says (in conditional world) : People who tend to trust others => do not believe that others distrust them.

Doubt:
Would it be okay if I modify the above sentence to say that :

People who do not believe that others distrust them => People who tend to trust others?


I am a bit confused about the direction of the arrow. I know that if the argument were "some people ..." then the direction of the arrow would change the whole game. But, would it be okay to switch the arrows without "some" etc...

NB:
I have drawn a picture to see how the assumption looks like. Please let me know whether it's correct
You do not have the required permissions to view the files attached to this post.
 Nikki Siclunov
PowerScore Staff
  • PowerScore Staff
  • Posts: 1362
  • Joined: Aug 02, 2011
|
#4847
First off, note that this is a Justify the Conclusion question, not an Assumption question (the conclusion follows logically if which of the following is assumed?).

Second, avoid using Venn diagrams to represent conditional relationships. It's too time-consuming and not appropriate for a standardized test, such as the LSAT.

The structure of this argument is as follows:

Premise: People who do NOT believe others distrust them :arrow: Confident

Premise: Confident :arrow: Regard as Challenge

Conclusion: Trust others :arrow: Regard as Challenge

In other words:

Premise: A :arrow: B
Premise: B :arrow: C

Conclusion: X :arrow: C

The link you need to establish is X :arrow: A. In the argument above, this link would sound like this:

People who trust others do NOT believe others distrust them (or the contrapositive). This prephrase agrees with answer choice (C).

Get the most out of your LSAT Prep Plus subscription.

Analyze and track your performance with our Testing and Analytics Package.